\documentclass[14pt,notheorems,leqno,xcolor={rgb}]{beamer}
\usepackage{ctex}
\usepackage{amsmath}
%\newtheorem{example}{例}
%\newtheorem{theorem}{定理}
%\newtheorem{remark}{注记}
%\newtheorem{corollary}{推论}
%\newtheorem{exercise}{练习}

\begin{document}
\begin{frame}
\frametitle{费马引理}
{费马引理}\quad
设$f(x)$在点$x_0$的某邻域$U(x_0)$内有定义,
且$\forall x\in U(x_0)$有$f(x)\le f(x_0)$(或$f(x)\ge f(x_0)$).
如果$f(x)$在$x_0$处可导.则有$f’(x_0)=0$.
\end{frame}
\end{document}

错误提示为:Font shape ‘OTl/cmss/m/it’ in sizeundefined

1 回答1

0
1. 我这编译没`error`,你说的可能是警告`warning` 1. 请将代码放入代码块

作者追问:2019-12-09 18:25

\documentclass[14pt,notheorems,leqno,xcolor={rgb}]{beamer} \usepackage{ctex} \usepackage{amsmath} %\newtheorem{example}{例} %\newtheorem{theorem}{定理} %\newtheorem{remark}{注记} %\newtheorem{corollary}{推论} %\newtheorem{exercise}{练习} \begin{document} \begin{frame} \frametitle{费马引理} {费马引理}\quad 设$f(x)$在点$x_0$的某邻域$U(x_0)$内有定义, 且$\forall x\in U(x_0)$有$f(x)\le f(x_0)$(或$f(x)\ge f(x_0)$). 如果$f(x)$在$x_0$处可导.则有$f'(x_0)=0$. \end{frame} \begin{frame} \frametitle{罗尔定理} \begin{theorem} 如果函数$f(x)$满足条件: \begin{enumskip} \item(1) 在闭区间$[a,b]$上连续, \item(2)在开区间$(a,b)$上可导, \item(3)在端点处$f(a)=f(b)$, \end{enumskip} 则至少存在一点$\xi\in(a,b)$,使得$f'(\xi)=0$. \end{theorem}\\ 如果定理的三个条件有一个不满足,则结论可能不成立. \end{frame} \end{document}

回答: 2019-12-09 18:30

你后面提供的代码,报错为`Environment theorem undefined.` ```tex \documentclass[14pt,notheorems,leqno,xcolor={rgb}]{beamer} \usepackage{ctex} \usepackage{amsmath} %\newtheorem{example}{例} %\newtheorem{theorem}{定理} %\newtheorem{remark}{注记} %\newtheorem{corollary}{推论} %\newtheorem{exercise}{练习} \begin{document} \begin{frame} \frametitle{费马引理} {费马引理}\quad 设$f(x)$在点$x_0$的某邻域$U(x_0)$内有定义, 且$\forall x\in U(x_0)$有$f(x)\le f(x_0)$(或$f(x)\ge f(x_0)$). 如果$f(x)$在$x_0$处可导.则有$f’(x_0)=0$. \end{frame} \begin{frame} \frametitle{罗尔定理} \begin{theorem} 如果函数$f(x)$满足条件: \begin{enumskip} \item(1) 在闭区间$[a,b]$上连续, \item(2)在开区间$(a,b)$上可导, \item(3)在端点处$f(a)=f(b)$, \end{enumskip} 则至少存在一点$\xi\in(a,b)$,使得$f’(\xi)=0$. \end{theorem}\\ 如果定理的三个条件有一个不满足,则结论可能不成立. \end{frame} \end{document} ``` 第21行和第29行中的`theorem`环境未定义,第23行和第27行中的`enumskip`环境未定义,不是自己的代码请不要抄,尤其是你不知道这段代码是什么意思的时候

回答: 2019-12-09 18:34

这段代码能运行,不知道是不是你想要的效果 ```tex \documentclass[14pt,notheorems,leqno,xcolor={rgb}]{beamer} \usepackage{ctex} \usepackage{amsmath} \newtheorem{theorem}{定理} \begin{document} \begin{frame} \frametitle{费马引理} {费马引理}\quad 设$f(x)$在点$x_0$的某邻域$U(x_0)$内有定义, 且$\forall x\in U(x_0)$有$f(x)\le f(x_0)$(或$f(x)\ge f(x_0)$). 如果$f(x)$在$x_0$处可导.则有$f’(x_0)=0$. \end{frame} \begin{frame} \frametitle{罗尔定理} \begin{theorem} 如果函数$f(x)$满足条件: \begin{enumerate} \item[(1)] 在闭区间$[a,b]$上连续, \item[(2)]在开区间$(a,b)$上可导, \item[(3)]在端点处$f(a)=f(b)$, \end{enumerate} 则至少存在一点$\xi\in(a,b)$,使得$f’(\xi)=0$. \end{theorem} 如果定理的三个条件有一个不满足,则结论可能不成立. \end{frame} \end{document} ```

作者追问:2019-12-09 18:47

能帮我看一下这个吗?实在没办法了 \documentclass[14pt,notheorems,leqno,xcolor={rgb}]{beamer} \usepackage{ctex} \usepackage{amsmath} \newtheorem{example}{例} \newtheorem{theorem}{定理} \newtheorem{remark}{注记} \newtheorem{corollary}{推论} \newtheorem{exercise}{练习} \begin{document} \begin{frame} \frametitle{费马引理} {费马引理}\quad 设$f(x)$在点$x_0$的某邻域$U(x_0)$内有定义, 且$\forall x\in U(x_0)$有$f(x)\le f(x_0)$(或$f(x)\ge f(x_0)$). 如果$f(x)$在$x_0$处可导.则有$f’(x_0)=0$. \end{frame} \begin{frame} \frametitle{罗尔定理} \begin{theorem} 如果函数$f(x)$满足条件: \begin{enumerate} \item[(1)]在闭区间$[a,b]$上连续, \item[(2)]在开区间$(a,b)$上可导, \item[(3)]在端点处$f(a)=f(b)$, \end{enumerate} 则至少存在一点$\xi\in(a,b)$,使得$f’(\xi)=0$. \end{theorem} 如果定理的三个条件有一个不满足,则结论可能不成立. \end{frame} \begin{frame} \begin{example} 下列函数只满足罗尔定理的条件(2)和(3),不满足条件(1),因此没有导数为零的点. \[f(x)=\begin{cases}x,&-1\leq x<1 x=1\end{cases}\] f(x)=|x|, f(x)=x, f(x)=x^2-2x-3$在区间$[-1,3]$上验证罗尔定理. f(x)=\dfrac1{1+x^2}$在区间$[-2,2]$上验证罗尔定理. f(x)$在$[0,1]$上连续,在$(0,1)$上可导,而且$f(0)=0$,$f(1) xi)=2\xi$. xi)=\frac{f(b)-f(a)}{b-a}$. f(x)=x^3$在区间$[0,1]$上验证拉格朗日定理. f(x)=x^3+x$在区间$[-1,1]$上验证拉格朗日定理.\\ xss=removed>x_1$时不等式成立: $$\arctan x_2-\arctan x_1 \le x_2-x_1.$$ \end{example} \begin{exercise} 证明:当$x_2>x_1$时有 $$\sin x_2-\sin x_1 \le x_2-x_1.$$ \end{exercise} \end{frame} \begin{frame} \begin{example} 证明当$x>0$时不等式成立: $$\dfrac{x}{1+x}<\ln(1+x)

回答: 2019-12-09 18:50

代码放代码块里,用法见 https://wenda.latexstudio.net/q-1474.html

作者追问:2019-12-09 22:23

就是这个,谢谢了,刚刚不会发。 ``` \documentclass[14pt,notheorems,leqno,xcolor={rgb}]{beamer} \usepackage{ctex} \usepackage{amsmath} \newtheorem{example}{例} \newtheorem{theorem}{定理} \newtheorem{remark}{注记} \newtheorem{corollary}{推论} \newtheorem{exercise}{练习} \begin{document} \begin{frame} \frametitle{费马引理} {费马引理}\quad 设$f(x)$在点$x_0$的某邻域$U(x_0)$内有定义, 且$\forall x\in U(x_0)$有$f(x)\le f(x_0)$(或$f(x)\ge f(x_0)$). 如果$f(x)$在$x_0$处可导.则有$f'(x_0)=0$. \end{frame} \begin{frame} \frametitle{罗尔定理} \begin{theorem} 如果函数$f(x)$满足条件: \begin{enumskip} \item(1) 在闭区间$[a,b]$上连续, \item(2)在开区间$(a,b)$上可导, \item(3)在端点处$f(a)=f(b)$, \end{enumskip} 则至少存在一点$\xi\in(a,b)$,使得$f'(\xi)=0$. \end{theorem}\\ 如果定理的三个条件有一个不满足,则结论可能不成立. \end{frame} \begin{frame} \begin{example} 下列函数只满足罗尔定理的条件(2)和(3),不满足条件(1),因此没有导数为零的点. \[f(x)=\begin{cases}x,&-1\leq x<1 x=1\end{cases}\] f(x)=|x|, f(x)=x, f(x)=x^2-2x-3$在区间$[-1,3]$上验证罗尔定理. f(x)=\dfrac1{1+x^2}$在区间$[-2,2]$上验证罗尔定理. f(x)$在$[0,1]$上连续,在$(0,1)$上可导,而且$f(0)=0$,$f(1) xi)=2\xi$. xi)=\frac{f(b)-f(a)}{b-a}$. f(x)=x^3$在区间$[0,1]$上验证拉格朗日定理. f(x)=x^3+x$在区间$[-1,1]$上验证拉格朗日定理.\\ xss=removed>x_1$时不等式成立: $$\arctan x_2-\arctan x_1 \le x_2-x_1.$$ \end{example} \begin{exercise} 证明:当$x_2>x_1$时有 $$\sin x_2-\sin x_1 \le x_2-x_1.$$ \end{exercise} \end{frame} \begin{frame} \begin{example} 证明当$x>0$时不等式成立: $$\dfrac{x}{1+x}<\ln(1+x)

回答: 2019-12-09 22:35

我不知道这段代码是谁给你的,麻烦你去看lshort-zh-cn [https://github.com/CTeX-org/lshort-zh-cn](https://github.com/CTeX-org/lshort-zh-cn),谢谢,我真的改的心累,为什么会缺`$`符号?为什么没定义环境加`*`? ```tex \documentclass[14pt,notheorems,leqno,xcolor={rgb}]{beamer} \usepackage{ctex} \usepackage{amsmath} \newtheorem{example}{例} \newtheorem{theorem}{定理} \newtheorem{remark}{注记} \newtheorem{corollary}{推论} \newtheorem{exercise}{练习} \begin{document} \begin{frame} \frametitle{费马引理} {费马引理}\quad 设$f(x)$在点$x_0$的某邻域$U(x_0)$内有定义, 且$\forall x\in U(x_0)$有$f(x)\le f(x_0)$(或$f(x)\ge f(x_0)$). 如果$f(x)$在$x_0$处可导.则有$f'(x_0)=0$. \end{frame} \begin{frame} \frametitle{罗尔定理} \begin{theorem} 如果函数$f(x)$满足条件: \begin{enumerate} \item(1) 在闭区间$[a,b]$上连续, \item(2)在开区间$(a,b)$上可导, \item(3)在端点处$f(a)=f(b)$, \end{enumerate} 则至少存在一点$\xi\in(a,b)$,使得$f'(\xi)=0$. \end{theorem} 如果定理的三个条件有一个不满足,则结论可能不成立. \end{frame} \begin{frame} \begin{example} 下列函数只满足罗尔定理的条件(2)和(3),不满足条件(1),因此没有导数为零的点. \[f(x)=\begin{cases}x,&-1\leq x<1 x=1\end{cases}\] f(x)=|x|, f(x)=x, f(x)=x^2-2x-3$在区间$[-1,3]$上验证罗尔定理. f(x)=\dfrac1{1+x^2}$在区间$[-2,2]$上验证罗尔定理. f(x)$在$[0,1]$上连续,在$(0,1)$上可导,而且$f(0)=0$,$f(1) xi)=2\xi$. xi)=\frac{f(b)-f(a)}{b-a}$. f(x)=x^3$在区间$[0,1]$上验证拉格朗日定理. f(x)=x^3+x$在区间$[-1,1]$上验证拉格朗日定理.\\ xss=removed>x_1$时不等式成立: $$\arctan x_2-\arctan x_1 \le x_2-x_1.$$ \end{example} \begin{exercise} 证明:当$x_2>x_1$时有 $$\sin x_2-\sin x_1 \le x_2-x_1.$$ \end{exercise} \end{frame} \begin{frame} \begin{example} 证明当$x>0$时不等式成立: $$\dfrac{x}{1+x}<\ln(1+x)
  • 回复 undefined :那就等你提供完整的代码 – sikouhjw 2019-12-09 19:15 回复
  • 没电脑,弄不了了。 – yuwei 2019-12-09 19:08 回复
  • 回复 undefined :那个链接写的不是挺清楚的吗,在你的代码前面加上```,代码后面加上``` – sikouhjw 2019-12-09 19:06 回复
  • 有点不太懂 – yuwei 2019-12-09 19:03 回复
  • 是的,谢谢 – yuwei 2019-12-09 18:38 回复
  • 回复 undefined :提供代码,我不明白你是怎么添加的 – sikouhjw 2019-12-09 18:24 回复
  • 我再添加一个\begin{frame} ... \end{frame} 就错了 – yuwei 2019-12-09 18:21 回复

你的回答

请登录后回答

你的回答将会帮助更多人,请务必认真回答问题。